3.I.1H

Number Theory | Part II, 2008

Prove that, for all x2x \geqslant 2, we have

px1p>loglogx12\sum_{p \leqslant x} \frac{1}{p}>\log \log x-\frac{1}{2}

[You may assume that, for 0<u<10<u<1,

log(1u)u<u22(1u).-\log (1-u)-u<\frac{u^{2}}{2(1-u)} .

Typos? Please submit corrections to this page on GitHub.